Cryptography problem involving prime numbers

Click For Summary
The discussion revolves around solving a cryptography problem involving prime numbers and modular arithmetic. The initial attempt shows that raising both sides to the power of (p-1)/2 leads to the conclusion that x^2 = -1 (mod p) has no integer solutions. The user struggles to incorporate Fermat's Little Theorem into their approach, which could help clarify the relationship between x^(p-1) and -1 (mod p). A suggestion is made to substitute x^(p-1) ≡ 1 (mod p) to draw a clearer conclusion. The thread highlights the challenges in applying theoretical concepts to practical problems in cryptography.
docnet
Messages
796
Reaction score
486
Homework Statement
I'm having trouble solving this problem. any help would be appreciated.
Relevant Equations
p = 3(mod4)
x^2 = -1(modp)
Screen Shot 2020-09-21 at 7.04.39 PM.png


Here is my attempt

When we raise both sides to the power (p-1)/2, we get
x^(p-1)= -1^[(p-1)/2](modp)

Looking at p=3(mod4), the possible values of p are
{3, 7, 11, 19, 23, 31...}.

Putting these values of p into (p-1)/2 we get odd integers.
{1, 3, 5, 9, 11, 15...}.

So we have
x^(p-1) = -1(modp)

We let p = 3 then we have
(x)^2 = -1(mod3)

which is equivalent to
x^2=3k -1

With the solutions
x= {sqrt(2), sqrt(5), sqrt(8), ...}

The solutions are not integers, which are undefined for the modulo operation. therefore x^2=-1(modp) have no solutions.

This is the wrong answer. I don't know
 
Physics news on Phys.org
It doesn't seem like you've tried using Fermat's little theorem yet.
 
Im struggling to see how to incorporate the information from FLT into the problem.
image.jpg
 
Can a moderator move this thread into the right subforum? Sorry, I should have posted this in the “calculus and beyond.“ section. Thanks
 
docnet said:
Homework Statement:: I'm having trouble solving this problem. any help would be appreciated.
Relevant Equations:: p = 3(mod4)
x^2 = -1(modp)

View attachment 269850

Here is my attempt

When we raise both sides to the power (p-1)/2, we get
x^(p-1)= -1^[(p-1)/2](modp)

Looking at p=3(mod4), the possible values of p are
{3, 7, 11, 19, 23, 31...}.

Putting these values of p into (p-1)/2 we get odd integers.
{1, 3, 5, 9, 11, 15...}.

So we have
x^(p-1) = -1(modp)

It would have been better to get here by setting p = 4k + 3 so that <br /> \frac{p - 1}{2} = \frac{4k + 2}{2} = 2k + 1 and hence (-1)^{(p-1)/2} = -1.
 
  • Like
Likes docnet
docnet said:
Im struggling to see how to incorporate the information from FLT into the problem.
View attachment 269890

You just need to substitute x^{p-1} \equiv 1 \mod p in <br /> x^{p-1} = -1 \mod p and draw the appropriate conclusion.
 
  • Like
Likes docnet
Question: A clock's minute hand has length 4 and its hour hand has length 3. What is the distance between the tips at the moment when it is increasing most rapidly?(Putnam Exam Question) Answer: Making assumption that both the hands moves at constant angular velocities, the answer is ## \sqrt{7} .## But don't you think this assumption is somewhat doubtful and wrong?

Similar threads

  • · Replies 1 ·
Replies
1
Views
2K
  • · Replies 6 ·
Replies
6
Views
2K
Replies
3
Views
1K
Replies
5
Views
2K
  • · Replies 2 ·
Replies
2
Views
3K
Replies
15
Views
4K
  • · Replies 16 ·
Replies
16
Views
3K
  • · Replies 2 ·
Replies
2
Views
4K
Replies
30
Views
3K
  • · Replies 2 ·
Replies
2
Views
1K